Vectors b and c Parallel? Find Scalar s or N for No

  • Thread starter Thread starter tarheels88
  • Start date Start date
  • Tags Tags
    Parallel Vectors
Click For Summary
Vectors b and c are evaluated for parallelism, with b=<0,3,4> and c=<0,6,-8> resulting in a conclusion of "No" since no scalar s satisfies c=sb. In contrast, for b=<10,6> and c=<-15,-9>, a scalar s of -3/2 confirms their parallelism. The discussion emphasizes the importance of explaining the reasoning behind the scalar choice, particularly in the first case. The conclusion is that while the first pair of vectors is not parallel, the second pair is, with the appropriate scalar identified. Clarity in the explanation of scalar selection is crucial for homework submissions.
tarheels88
Messages
8
Reaction score
0

Homework Statement


Given vectors b and c are they parallel? If so find such scalar s that c=sb, otherwise write N for no.

a) b=<0,3,4>, c=<0,6,-8)

b) b=<10,6>, c=<-15,-9>

Homework Equations



c=sb is a property of the dot product.

The Attempt at a Solution


For a) I put No because <0,6,-8> does not = <0s, 3s, 4s> because S=2
For b) I put Yes because S=-3/2 which would give you both outcomes of the vectors

Am I correct?
 
Physics news on Phys.org
Yes, you are. If you'd explain why you need to pick s=2 in the first case, that would probably help. "because s=2" isn't much of an explanation.
 
I put s=2 because I was finding a multiple of the two vectors given. Then I calculate 3s=6, that would then calculate to c=2.
 
tarheels88 said:
I put s=2 because I was finding a multiple of the two vectors given. Then I calculate 3s=6, that would then calculate to c=2.

Right, so the only possibility is s=2, and since that doesn't work there is no s that works. I knew you knew that. Just wanted you to add that if you are turning this in as a homework exercise.
 
Question: A clock's minute hand has length 4 and its hour hand has length 3. What is the distance between the tips at the moment when it is increasing most rapidly?(Putnam Exam Question) Answer: Making assumption that both the hands moves at constant angular velocities, the answer is ## \sqrt{7} .## But don't you think this assumption is somewhat doubtful and wrong?

Similar threads

  • · Replies 9 ·
Replies
9
Views
2K
  • · Replies 3 ·
Replies
3
Views
1K
Replies
11
Views
2K
Replies
1
Views
1K
  • · Replies 6 ·
Replies
6
Views
2K
Replies
26
Views
2K
Replies
2
Views
2K
  • · Replies 7 ·
Replies
7
Views
2K
  • · Replies 3 ·
Replies
3
Views
2K
  • · Replies 5 ·
Replies
5
Views
2K